[Physics] Time reversal symmetry of transverse field Ising model

ising-modelquantum mechanicssymmetrytime-reversal-symmetry

Is the transverse field Ising model time-reversal invariant? Specifically consider a non-integrable variant:

\begin{equation}
H = -J \sum_i^{L-1} \sigma_i^z \sigma_{i+1}^z + g \sum_i^L \sigma_i^x + h \sum_i^L \sigma_i^z,
\end{equation}

so that it has both a transverse and longitudinal field. $\sigma$s are the usual Pauli-matrices.

So, is it TRI?

Arguments for no:

1) The time-reversal operator is $T = K \prod_{j=1}^L (-i \sigma^y_j)$ where $K$ is complex conjugation in the $\uparrow,\downarrow$ basis. One can check that $[H,T] \neq 0$.

2) Simply recall the action of $T$ on spins. It flips $\vec{S} \to -\vec{S}$. So the transverse fields $\sigma^z$ and $\sigma^x$ flip, and the model is not invariant.

Arguments for yes:

1) One must recall that $g, h$ are external magnetic fields which are pseudovectors. So we have to flip them at the same time as flipping $\vec{S}$. So overall, yes, TRI.

2) The level statistics of the model obey well GOE (Gaussian orthogonal ensemble) statistics (see arxiv:1306.4306 for example), a class of random matrices that are supposed to describe time-reversal invariant Hamiltonians.

Puzzling issues:

1) If no, how to reconcile with GOE statistics?

2) If yes because of reason 1, that feels like it is cheating; we have to put in the way the constants $g,h$ transform by hand.

3) If yes, Kramers theorem says the spectrum should be doubly-degenerate for a chain of total spin half-integral. Quickly check in Mathematica for $L = 3,5,7\cdots$, this is untrue – the spectrum is not degenerate. Kramer's theorem doesn't hold.

So, yes or no? Or yes and no?

EDIT
Also, I've heard this phrase alot: oh, the Hamiltonian is real so it is time-reversal symmetric.

Does that make sense??

I mean, if I had applied my transverse field in the $y$-direction, I don't expect any physical difference. It's just a magnetic field pointing a different way. But in the particular basis I've chosen, the model is now complex. So it has become non-time-reversal symmetric?

Oh is the statement "a real Hamiltonian is TRI" too cavalier?

Best Answer

Basically, the answer is yes: $H$ is TRI because it is real. Reality condition really means that the Hamiltonian obeys a certain anti-unitary symmetry. In this case, the time-reversal operation is simply $T=K$ where $K$ is the complex conjugation. It is not the usual one($T=K\prod_i i\sigma^y_i$), and in particular $T^2=1$, so there is no Kramers' theorem and the spectrum is not doubly degenerate. The fact that level statistics follows GOE of course is a consequence of the reality condition. In fact, I think if there was a $T^2=-1$ time-reversal symmetry, the statistics would follow a different ensemble (the symplectic one, I believe).

You asked what if one changes the transverse field $g\sum_i \sigma^x_i$ to the $y$ direction. In that case, the two Hamiltonians are unitarily related (i.e. a $\pi/2$ spin rotation $U_z$ around $z$ would bring it back). Let me call the Hamiltonian with $x$ transverse field $H_x(g)$ where $g$ is the transverse field, and with $y$ transverse field $H_y(g)$. Define $U_z=e^{i\pi \sum_i\sigma^z_i/4}$, then it is easy to check that $U_z H_y(g) U_z^{-1}= H_x(g)$. Since we know $H_x^*(g)=H_x(g)$, we can easily find $H_y^*=U_z^2 H_y U_z^{-2}$. Therefore one just has to redefine the time-reversal symmetry to be $T=K U_z^2$. If you really want to break the reality condition, in a way that can not be fixed by additional unitary transformations, then one needs to turn on transverse fields along all three dimensions.

Last comment on your "Arguments for yes": the first argument you gave, namely one also flips the external parameters, does not work. In this way, there would be no time-reversal symmetry breaking, except the CP violation in the fundamental processes! When we talk about the symmetry of a Hamiltonian, we should just treat the system on its own, not with all the external devices that generate the various terms -- unless you want to consider the dynamics of these devices, but then it is a different Hamiltonian.